2015 AMC 8

Complete problem set with solutions and individual problem pages

Problem 1 Easy

Onkon wants to cover his room's floor with his favourite red carpet. How many square yards of red carpet are required to cover a rectangular floor that is 12 feet long and 9 feet wide? (There are 3 feet in a yard.)

  • A.

    12

  • B.

    36

  • C.

    108

  • D.

    324

  • E.

    972

Answer:A

Solution 1

We need 12\cdot9 square feet of carpet to cover the floor. Since there are 9 square feet in a square yard, we divide this by 9 to get {\boxed{\textbf{(A) }12}} square yards.

 

Solution 2

Since there are 3 feet in a yard, we divide 9 by 3 to get 3 yards, and 12 by 3 to get 4 yards. To find the area of the carpet, we then multiply these two values together to get 3\cdot4=\boxed{\textbf{(A) }12} square yards.

Link Problem
Problem 2 Easy

Point O is the center of the regular octagon ABCDEFGH, and X is the midpoint of the side \overline{AB}. What fraction of the area of the octagon is shaded?

  • A.

    \frac{11}{32}

  • B.

    \frac{3}{8}

  • C.

    \frac{13}{32}

  • D.

    \frac{7}{16}

  • E.

    \frac{15}{32}

Answer:D

Solution 1

Since octagon ABCDEFGH is a regular octagon, it is split into 8 equal parts, such as triangles \bigtriangleup ABO, \bigtriangleup BCO, \bigtriangleup CDO, etc. These parts, since they are all equal, are \frac{1}{8} of the octagon each. The shaded region consists of 3 of these equal parts plus half of another, so the fraction of the octagon that is shaded is \frac{1}{8}+\frac{1}{8}+\frac{1}{8}+\frac{1}{16}=\boxed{\textbf{(D) }\dfrac{7}{16}}.

 

Solution 2

The octagon has been divided up into 16 identical triangles (and thus they each have equal area). Since the shaded region occupies 7 out of the 16 total triangles, the answer is \boxed{\textbf{(D)}~\dfrac{7}{16}}.

Link Problem
Problem 3 Easy

Jack and Jill are going swimming at a pool that is one mile from their house. They leave home simultaneously. Jill rides her bicycle to the pool at a constant speed of 10 miles per hour. Jack walks to the pool at a constant speed of 4 miles per hour. How many minutes before Jack does Jill arrive?

  • A.

    5

  • B.

    6

  • C.

    8

  • D.

    9

  • E.

    10

Answer:D

Using d=rt, we can set up an equation for when Jill arrives at the swimming pool:

1=10t

Solving for t, we get that Jill gets to the pool in \frac{1}{10} of an hour, which is 6 minutes. Doing the same for Jack, we get that

Jack arrives at the pool in \frac{1}{4} of an hour, which in turn is 15 minutes. Thus, Jill has to wait 15-6=\boxed{\textbf{(D)}~9}

Link Problem
Problem 4 Easy

The Blue Bird High School chess team consists of two boys and three girls. A photographer wants to take a picture of the team to appear in the local newspaper. She decides to have them sit in a row with a boy at each end and the three girls in the middle. How many such arrangements are possible?

  • A.

    2

  • B.

    4

  • C.

    5

  • D.

    6

  • E.

    12

Answer:E

There are 2! = 2 ways to order the boys on the ends, and there are 3!=6 ways to order the girls in the middle. We get the answer to be 2 \cdot 6 = \boxed{\textbf{(E) }12}.

Link Problem
Problem 5 Easy

Billy's basketball team scored the following points over the course of the first 11 games of the season. If his team scores 40 in the 12^{th} game, which of the following statistics will show an increase?

42, 47, 53, 53, 58, 58, 58, 61, 64, 65, 73

  • A.

    \text{range}

  • B.

    \text{median}

  • C.

    \text{mean}

  • D.

    \text{mode}

  • E.

    \text{mid-range}

Answer:A

Solution 1

When they score a 40 on the next game, the range increases from 73-42=31 to 73-40=33. This means the \boxed{\textbf{(A) } \text{range}} increased.

Note: The range is defined to be the difference of the largest element and the smallest element of a set.

 

Solution 2

Because 40 is less than the score of every game they've played so far, the measures of center will never rise. Only measures of spread, such as the \boxed{\textbf{(A)}~\text{range}}, may increase.

Link Problem
Problem 6 Easy

In \bigtriangleup ABC, AB=BC=29, and AC=42. What is the area of \bigtriangleup ABC?

  • A.

    100

  • B.

    420

  • C.

    500

  • D.

    609

  • E.

    701

Answer:B

Solution 1

We know the semi-perimeter of \triangle ABC is \frac{29+29+42}{2}=50. Next, we use Heron's Formula to find that the area of the triangle is just \sqrt{50(50-29)^2(50-42)}=\sqrt{50 \cdot 21^2 \cdot 8}=\boxed{\textbf{(B) }420}.

 

Solution 2 

Splitting the isosceles triangle in half, we get a right triangle with hypotenuse 29 and leg 21. Using the Pythagorean Theorem , we know the height is \sqrt{29^2-21^2}=20. Now that we know the height, the area is \dfrac{(20)(42)}{2} = \boxed{\textbf{(B)}~420}.

Link Problem
Problem 7 Easy

Each of two boxes contains three chips numbered 1, 2, 3. A chip is drawn randomly from each box and the numbers on the two chips are multiplied. What is the probability that their product is even?

  • A.

    \frac19

  • B.

    \frac29

  • C.

    \frac49

  • D.

    \frac12

  • E.

    \frac59

Answer:E

Solution 1

You can make this problem into a spinner problem. You have the first spinner with 3 equally divided

sections: 1, 2, and 3. You make a second spinner that is identical to the first, with 3 equal sections of

1,2, and 3. If the first spinner lands on 1, it must land on two for the result to be even. You write down the first

combination of numbers: (1,2). Next, if the spinner lands on 2, it can land on any number on the second

spinner. We now have the combinations of (1,2) ,(2,1), (2,2), and (2,3). Finally, if the first spinner ends on 3, we

have (3,2). Since there are 3\cdot3=9 possible combinations, and we have 5 evens, the final answer is

\boxed{\textbf{(E) }\frac{5}{9}}.

 

Solution 2

We can list out the numbers. Box A has chips 1, 2, and 3, and Box B also has chips 1, 2, and 3. Chip 1 (from Box A)

could be with 3 partners from Box B. This is also the same for chips 2 and 3 from Box A. 3+3+3=9 total sums. Chip 1 could be multiplied with 2 other chips to make an even product, just like chip 3. Chip 2 can only multiply with 1 chip. 2+2+1=5. The answer is \boxed{\textbf{(E) }\frac{5}{9}}.

Link Problem
Problem 8 Easy

What is the smallest whole number larger than the perimeter of any triangle with a side of length 5 and a side of length 19?

  • A.

    24

  • B.

    29

  • C.

    43

  • D.

    48

  • E.

    57

Answer:D

Solution 1

We know from the Triangle Inequality that the last side, s, fulfills s<5+19=24. Adding 5+19 to both sides of the inequality, we get s+5+19<48, and because s+5+19 is the perimeter of our triangle, \boxed{\textbf{(D)}\ 48} is our answer.

 

Solution 2

s, fulfills s<5+19=24, meaning that s<24. The greatest value of s can be 23. 23+5+19 is 47. 48 is our nearest answer choice so its \boxed{\textbf{(D)}\ 48}.

Link Problem
Problem 9 Easy

On her first day of work, Janabel sold one widget. On day two, she sold three widgets. On day three, she sold five widgets, and on each succeeding day, she sold two more widgets than she had sold on the previous day. How many widgets in total had Janabel sold after working 20 days?

  • A.

    39

  • B.

    40

  • C.

    210

  • D.

    400

  • E.

    401

Answer:D

Solution 1

First, we have to find how many widgets she makes on Day 20. We can write the linear equation y=-1+2x to represent this situation. Then, we can plug in 20 for x: y=-1+2(20) -- y=-1+40 -- y=39. The sum of 1,3,5, \cdots 39 is \dfrac{(1 + 39)(20)}{2}= \boxed{\textbf{(D)}~400}.

 

Solution 2

The sum is just the first 20 odd counting/natural numbers, which is 20^2=\boxed{\textbf{(D)}~400}.

Link Problem
Problem 10 Easy

How many integers between 1000 and 9999 have four distinct digits?

  • A.

    3024

  • B.

    4536

  • C.

    5040

  • D.

    6480

  • E.

    6561

Answer:B

There are 9 choices for the first number, since it cannot be 0, there are only 9 choices left for the second number since it must differ from the first, 8 choices for the third number, since it must differ from the first two, and 7 choices for the fourth number, since it must differ from all three. This means there are 9 \times 9 \times 8 \times 7=\boxed{\textbf{(B) }4536} integers between 1000 and 9999 with four distinct digits.

Link Problem
Problem 11 Medium

In the small country of Mathland, all automobile license plates have four symbols. The first must be a vowel (A, E, I, O, or U), the second and third must be two different letters among the 21 non-vowels, and the fourth must be a digit (0 through 9). If the symbols are chosen at random subject to these conditions, what is the probability that the plate will read "AMC8"?

  • A.

    \frac{1}{22,050}

  • B.

    \frac{1}{21,000}

  • C.

    \frac{1}{10,500}

  • D.

    \frac{1}{2,100}

  • E.

    \frac{1}{1,050}

Answer:B

Solution 1

There is one favorable case, which is the license plate says "AMC8." We must now find how many total cases there are. There are 5 choices for the first letter (since it must be a vowel), 21 choices for the second letter (since it must be of 21 consonants), 20 choices for the third letter (since it must differ from the second letter), and 10 choices for the digit. This leads to 5 \cdot 21 \cdot 20 \cdot 10=21,000 total possible license plates. Therefore, the probability of a license plate saying "AMC8" is \boxed{\textbf{(B) } \frac{1}{21,000}}.

 

Solution 2

The probability of choosing A as the first letter is \dfrac{1}{5}. The probability of choosing M next is \dfrac{1}{21}. The probability of choosing C as the third letter is \dfrac{1}{20} (since there are 20 other consonants to choose from other than M). The probability of having 8 as the last number is \dfrac{1}{10}. We multiply all these to obtain \dfrac{1}{5}\cdot \dfrac{1}{21} \cdot \dfrac{1}{20}\cdot \dfrac{1}{10}=\dfrac{1}{5\times 21\times 20\times 10}=\dfrac{1}{21\times 100\times 10}=\boxed{\textbf{(B)}~\dfrac{1}{21,000}}.

Link Problem
Problem 12 Medium

How many pairs of parallel edges, such as \overline{AB} and \overline{GH} or \overline{EH} and \overline{FG}, does a cube have?

  • A.

    6

  • B.

    12

  • C.

    18

  • D.

    24

  • E.

    36

Answer:C

Solution 1

We first count the number of pairs of parallel lines that are in the same direction as \overline{AB}. The pairs of parallel lines are \overline{AB}\text{ and }\overline{EF}, \overline{CD}\text{ and }\overline{GH}, \overline{AB}\text{ and }\overline{CD}, \overline{EF}\text{ and }\overline{GH}, \overline{AB}\text{ and }\overline{GH}, and \overline{CD}\text{ and }\overline{EF}. These are 6 pairs total. We can do the same for the lines in the same direction as \overline{AE} and \overline{AD}. This means there are 6\cdot 3=\boxed{\textbf{(C) } 18} total pairs of parallel lines.

 

Solution 2

Look at any edge, let's say \overline{AB}. There are three ways we can pair \overline{AB} with another edge. \overline{AB}\text{ and }\overline{EF}, \overline{AB}\text{ and }\overline{HG}, and \overline{AB}\text{ and }\overline{DC}. There are 12 edges on a cube. 3 times 12 is 36. We have to divide by 2 because every pair is counted twice, so \frac{36}{2} is \boxed{\textbf{(C) } 18} total pairs of parallel lines.

Link Problem
Problem 13 Medium

How many subsets of two elements can be removed from the set \{1, 2, 3, 4, 5, 6, 7, 8, 9, 10, 11\} so that the mean (average) of the remaining numbers is 6?

  • A.

    1

  • B.

    2

  • C.

    3

  • D.

    5

  • E.

    6

Answer:D

Solution 1

Since there will be 9 elements after removal, and their mean is 6, we know their sum is 54. We also know that the sum of the set pre-removal is 66. Thus, the sum of the 2 elements removed is 66-54=12. There are only \boxed{\textbf{(D)}~5} subsets of 2 elements that sum to 12: \{1,11\}, \{2,10\}, \{3, 9\}, \{4, 8\}, \{5, 7\}.

 

Solution 2

We can simply remove 5 subsets of 2 numbers while leaving only 6 behind. The average of this one-number set is still 6, so the answer is \boxed{\textbf{(D)}~5}.

Link Problem
Problem 14 Medium

Which of the following integers cannot be written as the sum of four consecutive odd integers?

  • A.

    16

  • B.

    40

  • C.

    72

  • D.

    100

  • E.

    200

Answer:D

Solution 1

Let our 4 numbers be n, n+2, n+4, n+6, where n is odd. Then, our sum is 4n+12. The only answer choice that cannot be written as 4n+12, where n is odd, is \boxed{\textbf{(D)} 100}.

 

Solution 2

If the four consecutive odd integers are 2n-3,~ 2n-1, ~2n+1 and 2n+3; then, the sum is 8n. All the integers are divisible by 8 except \boxed{\textbf{(D)}~100}.

Link Problem
Problem 15 Medium

At Euler Middle School, 198 students voted on two issues in a school referendum with the following results: 149 voted in favor of the first issue and 119 voted in favor of the second issue. If there were exactly 29 students who voted against both issues, how many students voted in favor of both issues?

  • A.

    49

  • B.

    70

  • C.

    79

  • D.

    99

  • E.

    149

Answer:D

Solution 1

Let:

- A be the number of students who voted in favor of the first issue,

- B be the number of students who voted in favor of the second issue,

- A \cap B be the number of students who voted in favor of both issues.

We are given:

- A = 149

- B = 119

- 29 students voted against both issues, so the number of students who voted for at least one issue is:

198 - 29 = 169

By the principle of inclusion and exclusion:

A \cup B = A + B - A \cap B

Substitute known values:

169 = 149 + 119 - A \cap B

169 = 268 - A \cap B

A \cap B = 268 - 169 = \boxed{\textbf{(D) }99}

 

Solution 2

We can see that this is a Venn Diagram Problem.

First, we analyze the information given. There are 198 students. Let's use A as the first issue and B as the second issue.

149 students were for A, and 119 students were for B. There were also 29 students against both A and B.

Solving this without a Venn Diagram, we subtract 29 away from the total, 198. Out of the remaining 169 , we have 149 people for A and

119 people for B. We add this up to get 268 . Since that is more than what we need, we subtract 169 from 268 to get

\boxed{\textbf{(D)}~99}.

Link Problem
Problem 16 Hard

In a middle-school mentoring program, a number of the sixth graders are paired with a ninth-grade student as a buddy. No ninth grader is assigned more than one sixth-grade buddy. If \frac{1}{3} of all the ninth graders are paired with \frac{2}{5} of all the sixth graders, what fraction of the total number of sixth and ninth graders have a buddy?

  • A.

    \frac{2}{15}

  • B.

    \frac{4}{11}

  • C.

    \frac{11}{30}

  • D.

    \frac{3}{8}

  • E.

    \frac{11}{15}

Answer:B

Solution 1 

Let the number of sixth graders be s, and the number of ninth graders be n. Thus, \frac{n}{3}=\frac{2s}{5}, which simplifies to n=\frac{6s}{5}. Since we are trying to find the value of \frac{\frac{n}{3}+\frac{2s}{5}}{n+s}, we can just substitute \frac{6s}{5} for n into the equation. We then get a value of \frac{\frac{\frac{6s}{5}}{3}+\frac{2s}{5}}{\frac{6s}{5}+s} = \frac{\frac{6s+6s}{15}}{\frac{11s}{5}} = \frac{\frac{4s}{5}}{\frac{11s}{5}} = \boxed{\textbf{(B)}~\frac{4}{11}}.

 

Solution 2

We see that the minimum number of ninth graders is 6, because if there are 3 then there is 1 ninth-grader with a buddy, which would mean there are 2.5 sixth graders, which is impossible (of course unless you really do have half of a person). With 6 ninth-graders, 2 of them are in the buddy program, so there \frac{2}{\tfrac{2}{5}}=5 sixth-graders total, two of whom have a buddy. Thus, the desired fraction is \frac{2+2}{5+6}=\boxed{\textbf{(B) }\frac{4}{11}}.

Link Problem
Problem 17 Hard

Jeremy's father drives him to school in rush hour traffic in 20 minutes. One day there is no traffic, so his father can drive him 18 miles per hour faster and gets him to school in 12 minutes. How far in miles is it to school?

  • A.

    4

  • B.

    6

  • C.

    8

  • D.

    9

  • E.

    12

Answer:D

Solution 1

For starters, we identify d as distance and v as velocity (speed)

Writing the equation gives us: \frac{d}{v}=\frac{1}{3} and \frac{d}{v+18}=\frac{1}{5}.

This gives d=\frac{1}{5}v+3.6=\frac{1}{3}v, which gives v=27, which then gives d=\boxed{\textbf{(D)}~9}.

 

Solution 2

d = rt,d=frac{1}{3}times r = frac{1}{5}times (r + 18)$

\frac{r}{3} = \frac{r}{5} + \frac{18}{5}

10r = 270 so r = 27, plug into the first one and it's \boxed{\textbf{(D)}~9} miles to school.

Link Problem
Problem 18 Hard

An arithmetic sequence is a sequence in which each term after the first is obtained by adding a constant to the previous term. For example, 2,5,8,11,14 is an arithmetic sequence with five terms, in which the first term is 2 and the constant added is 3. Each row and each column in this 5\times5 array is an arithmetic sequence with five terms. The square in the center is labelled X as shown. What is the value of X?

  • A.

    21

  • B.

    31

  • C.

    36

  • D.

    40

  • E.

    42

Answer:B

Solution 1

We begin filling in the table. The top row has a first term 1 and a fifth term 25, so we have the common difference is \frac{25-1}4=6. This means we can fill in the first row of the table:

The fifth row has a first term of 17 and a fifth term of 81, so the common difference is \frac{81-17}4=16. We can fill in the fifth row of the table as shown:

We must find the third term of the arithmetic sequence with a first term of 13 and a fifth term of 49. The common difference of this sequence is \frac{49-13}4=9, so the third term is 13+2\cdot 9=\boxed{\textbf{(B) }31}.

 

Solution 2

The middle term of the first row is \frac{25+1}{2}=13, since the middle number is just the average in an arithmetic sequence. Similarly, the middle of the bottom row is \frac{17+81}{2}=49. Applying this again for the middle column, the answer is \frac{49+13}{2}=\boxed{\textbf{(B)}~31}.

Link Problem
Problem 19 Hard

A triangle with vertices as A=(1,3), B=(5,1), and C=(4,4) is plotted on a 6\times5 grid. What fraction of the grid is covered by the triangle?

  • A.

    \frac 16

  • B.

    \frac 15

  • C.

    \frac 14

  • D.

    \frac13

  • E.

    \frac12

Answer:A

Solution 1

The area of \triangle ABC is equal to half the product of its base and height. By the Pythagorean Theorem, we find its height is \sqrt{1^2+2^2}=\sqrt{5}, and its base is \sqrt{2^2+4^2}=\sqrt{20}. We multiply these and divide by 2 to find the area of the triangle is \frac{\sqrt{5 \cdot 20}}2=\frac{\sqrt{100}}2=\frac{10}2=5. Since the grid has an area of 30, the fraction of the grid covered by the triangle is \frac 5{30}=\boxed{\textbf{(A) }\frac{1}{6}}.

 

Solution 2

Note angle \angle ACB is right; thus, the area is \sqrt{1^2+3^2} \times \sqrt{1^2+3^2}\times \dfrac{1}{2}=10 \times \dfrac{1}{2}=5; thus, the fraction of the total is \dfrac{5}{30}=\boxed{\textbf{(A)}~\dfrac{1}{6}}.

Link Problem
Problem 20 Hard

Ralph went to the store and bought 12 pairs of socks for a total of $24. Some of the socks he bought cost $1 a pair, some of the socks he bought cost $3 a pair, and some of the socks he bought cost $4 a pair. If he bought at least one pair of each type, how many pairs of $1 socks did Ralph buy?

  • A.

    4

  • B.

    5

  • C.

    6

  • D.

    7

  • E.

    8

Answer:D

Solution 1

So, let there be x pairs of $1 socks, y pairs of $3 socks, and z pairs of $4 socks.

We have x+y+z=12, x+3y+4z=24, and x,y,z \geqslant 1.

Now, we subtract to find 2y+3z=12, and y,z \geqslant 1. It follows that 2y is a multiple of 3 and 3z is a multiple of 3. Since sum of 2 multiples of 3 = multiple of 3, so we must have 2y=6.

Therefore, y=3, and it follows that z=2. Now, x=12-y-z=12-3-2=\boxed{\textbf{(D)}~7}, as desired.

 

Solution 2

Since the total cost of the socks was $24 and Ralph bought 12 pairs, the average cost of each pair of socks is \frac{$24}{12} = $2.

There are two ways to make packages of socks that average to $2. You can have:

\bullet Two $1 pairs and one $4 pair (package adds up to $6)

\bullet One $1 pair and one $3 pair (package adds up to $4)

Now, we need to solve

6a+4b=24,

where a is the number of $6 packages and b is the number of $4 packages. We see our only solution (that has at least one of each pair of sock) is a=2, b=3, which yields the answer of 2\times2+3\times1 = \boxed{\textbf{(D)}~7}.

Link Problem
Problem 21 Hard

In the given figure, hexagon ABCDEF is equiangular, ABJI and FEHG are squares with areas 18 and 32 respectively, \triangle JBK is equilateral and FE=BC. What is the area of \triangle KBC?

  • A.

    6 \sqrt2

  • B.

    9

  • C.

    12

  • D.

    9 \sqrt2

  • E.

    32

Answer:C

Solution 1

Clearly, since \overline{FE} is a side of a square with area 32, \overline{FE} = \sqrt{32} = 4 \sqrt{2}. Now, since \overline{FE} = \overline{BC}, we have \overline{BC} = 4 \sqrt{2}.

We know that \overline{JB} is a side of a square with area 18, so \overline{JB} = \sqrt{18} = 3 \sqrt{2}. Since \triangle JBK is equilateral, \overline{BK} = \overline{JB} = 3 \sqrt{2}.

Lastly, \triangle KBC is a right triangle. We see that \angle JBA + \angle ABC + \angle CBK + \angle KBJ = 360^{\circ} \rightarrow 90^{\circ} + 120^{\circ} + \angle CBK + 60^{\circ} = 360^{\circ} \rightarrow \angle CBK = 90^{\circ}, so \triangle KBC is a right triangle with legs 3 \sqrt{2} and 4 \sqrt{2}. Now, its area is \dfrac{3 \sqrt{2} \cdot 4 \sqrt{2}}{2} = \dfrac{24}{2} = \boxed{\textbf{(C)}~12}.

 

Solution 2

Since \overline{FE} = \sqrt{32}, and \overline{FE} = \overline{BC}, \overline{BC} = 4\sqrt{2}. Meanwhile, \overline{JB} = 3\sqrt{2}, and since \triangle JBK is equilateral, \overline{BK} = 3\sqrt{2}. If ABCDEF is equiangular, \angle ABC = \frac{180 \cdot (n-2)}{n} = 120^{\circ}, where n is the number of sides of the shape. Adding all the angles around B gives 270^{\circ}, so \angle KBC = 360 - 90 = 270^{\circ}. Because \triangle KBC is right, the area of \triangle KBC = \frac{4\sqrt{2} \cdot 3\sqrt{2}} {2} = 12. Therefore, the answer is \boxed{\textbf{(C)}~12}.

Link Problem
Problem 22 Hard

On June 1, a group of students is standing in rows, with 15 students in each row. On June 2, the same group is standing with all of the students in one long row. On June 3, the same group is standing with just one student in each row. On June 4, the same group is standing with 6 students in each row. This process continues through June 12 with a different number of students per row each day. However, on June 13, they cannot find a new way of organizing the students. What is the smallest possible number of students in the group?

  • A.

    21

  • B.

    30

  • C.

    60

  • D.

    90

  • E.

    1080

Answer:C

Solution 1

Since we know the number must be a multiple of 15, we can eliminate A. We also know that after 12 days, the students can't find any more arrangements, meaning the number has 12 factors. Now, we just list the factors of every number, starting with 30:

30=1\cdot30, 2\cdot15, 3\cdot10, 5\cdot6

60=1\cdot60, 2\cdot30, 3\cdot20, 4\cdot15, 5\cdot12, 6\cdot1060 has 12 factors, so the answer is \boxed{\textbf{(C) } 60}.

 

Solution 2

We know that the number of students in the group has to be a multiple of 15, so we can eliminate A. However, on June 13, they cannot find a new way of organizing the students meaning that the number has only 12 factors. The prime factorization of 60 is equal to 2^2 x 3^1 x 5^1. Using the factor counting formula, we have the add one to each exponent and multiply them together to find the number of factors. (2+1) \times (1+1) \times (1+1) = 3 \times 2 \times 2 = 12 factors making \boxed{\textbf{(C) } 60} the answer.

Link Problem
Problem 23 Hard

Tom has twelve slips of paper which he wants to put into five cups labeled A, B, C, D, E. He wants the sum of the numbers on the slips in each cup to be an integer. Furthermore, he wants the five integers to be consecutive and increasing from A to E. The numbers on the papers are 2, 2, 2, 2.5, 2.5, 3, 3, 3, 3, 3.5, 4, and 4.5. If a slip with 2 goes into cup E and a slip with 3 goes into cup B, then the slip with 3.5 must go into what cup?

  • A.

    A

  • B.

    B

  • C.

    C

  • D.

    D

  • E.

    E

Answer:D

The numbers have a sum of 6+5+12+4+8=35, which averages to 7, which means A, B, C, D, and E will have the values 5, 6, 7, 8 and 9, respectively. Now, it's the process of elimination: Cup A will have a sum of 5, so putting a 3.5 slip in the cup will leave 5-3.5=1.5; however, all of our slips are bigger than 1.5, so this is impossible. Cup B has a sum of 6, but we are told that it already has a 3 slip, leaving 6-3=3, which is too small for the 3.5 slip. Cup C is a little bit trickier but still manageable. It must have a value of 7, so adding the 3.5 slip leaves room for 7-3.5=3.5. This looks good at first as we have slips smaller than that, but upon closer inspection, we see that no slip fits exactly. And the smallest sum of two slips is 2+2=4, which is too big, so this case is also impossible. Cup E has a sum of 9, but we are told it already has a 2 slip. So, we are left with 9-2=7, which is identical to Cup C and thus also impossible. With all other choices removed, we are left with the answer: Cup \boxed{\textbf{(D)}~D}.

Link Problem
Problem 24 Hard

A baseball league consists of two four-team divisions. Each team plays every other team in its division N games. Each team plays every team in the other division M games with N>2M and M>4. Each team plays a 76 game schedule. How many games does a team play within its own division?

  • A.

    36

  • B.

    48

  • C.

    54

  • D.

    60

  • E.

    72

Answer:B

Solution 1

On one team they play 3N games in their division and 4M games in the other. This gives 3N+4M=76.

Since M>4 we start by trying M=5. This doesn't work because 56 is not divisible by 3.

Next, M=6 does not work because 52 is not divisible by 3.

We try M=7 does work by giving N=16 ,~M=7 and thus 3\times 16=\boxed{\textbf{(B)}~48} games in their division.

M=10 seems to work, until we realize this gives N=12, but N>2M so this will not work.

 

Solution 2

76=3N+4M > 10M, giving M \leqslant 7. Since M>4, we have M=5,6,7. Since 4M is 1 \pmod{3}, we must have M equal to 1 \pmod{3}, so M=7.

This gives 3N=48, as desired. The answer is \boxed{\textbf{(B)}~48}.

Link Problem
Problem 25 Hard

One-inch squares are cut from the corners of this 5 inch square. What is the area in square inches of the largest square that can be fitted into the remaining space?

  • A.

    9

  • B.

    12 \frac 12

  • C.

    15

  • D.

    15 \frac 12

  • E.

    17

Answer:C

We draw a square as shown:

We want to find the area of the biggest square. The area of this square is composed of the center white square and the four red triangles. Because the inner square has an area of (5-2), 3, squared, 9, it also has a length of \sqrt{9}=3. The heights of each of the red triangles are 1 (because the gray squares have lengths of one), and the area of one triangle is namely \frac{3 \cdot 1}{2}. Thus, the combined area of the four triangles is 4 \cdot \frac 32=6. Furthermore, the area of the smaller square is 9. We add these to see that the area of the large square is 9+6=15\implies \boxed{\textbf{(C)}}.

Link Problem
Table of Contents
  • 1
  • 2
  • 3
  • 4
  • 5
  • 6
  • 7
  • 8
  • 9
  • 10
  • 11
  • 12
  • 13
  • 14
  • 15
  • 16
  • 17
  • 18
  • 19
  • 20
  • 21
  • 22
  • 23
  • 24
  • 25